• Anúncio Global
    Respostas
    Exibições
    Última mensagem

valores das constantes a e b

valores das constantes a e b

Mensagempor Douglas16 » Dom Mar 31, 2013 16:36

Determinar os valores das constantes a e b de tal forma que

\lim_{x\rightarrow\frac{\pi}{2}}=\frac{-cos\left(2x \right)+asen\left(x \right)+b}{{\left(x-\frac{\pi}{2} \right)}^{4}} exista.

Depois, calcular o limite.

A única conclusão ou informação que consegui até agora é que b=-(a+1), isso vem do fato que para o valor limite existir, uma das condições é que tanto o limite do denominador quanto do numerador devem ser igual a zero, e a outra condição é a expressão seja tal que através do \lim_{x\rightarrow0} \frac{sen\left(x \right)}{x}=1 eu possa eliminar a indeterminação \frac{0}{0}, ou seja eliminar {\left(x-\frac{\pi}{2} \right)}^{4}.

Mas não vejo uma forma de fazer isso.
Tentei fazer a substituição: t=\left(x-\frac{\pi}{2} \right), mas ainda não vejo uma saída.
Douglas16
Usuário Parceiro
Usuário Parceiro
 
Mensagens: 69
Registrado em: Seg Fev 11, 2013 19:15
Formação Escolar: ENSINO MÉDIO
Área/Curso: Matemática
Andamento: cursando

Re: valores das constantes a e b

Mensagempor Douglas16 » Dom Mar 31, 2013 19:31

A expressão do numerador pode ser fatorada como: 2*sen{\left(x-1 \right)}^{2}, admitindo x=-4.
Douglas16
Usuário Parceiro
Usuário Parceiro
 
Mensagens: 69
Registrado em: Seg Fev 11, 2013 19:15
Formação Escolar: ENSINO MÉDIO
Área/Curso: Matemática
Andamento: cursando

Re: valores das constantes a e b

Mensagempor e8group » Dom Mar 31, 2013 20:23

Considere : x - \pi/2 = k quando x\to \pi/2 , k\to 0 .

O limite a ser calculado será então : \lim_{k \to 0} \frac{-cos(2k +\pi) + asin(k + \pi/2) + b}{k^4} ou \lim_{k \to 0}  \frac{cos(2k) +a \cdot cos(k)+b }{k^2} ou \lim_{k \to 0} \frac{cos^2(k) -  sin^2(k) + acos(k) + b}{k^2} ou ainda , \lim_{k \to 0} \frac{1 + b +  acos(k) -  2sin^2(k)  }{k^2} .

Mas, cos(k) = cos(2 \cdot k/2) = cos^2(k/2) - sin^2(k/2) = 1 - 2sin^2(k/2) , então :

\lim_{k \to 0} \frac{1 + b +  acos(k) -  2sin^2(k)  }{k^2} = \lim_{k \to 0} \frac{1 + b +  a - 2asin^2(k/2) -  2sin^2(k)  }{k^2}



vemos então que o limite existe se ,e somente se , 1 + b +  a = 0 (Por quê ?)

Ou seja ,dado um a (ou b) real ,temos que b = -(1+a) (ou a = -(1+b) .) .Nestas condições o limite existirá .
e8group
Colaborador Voluntário
Colaborador Voluntário
 
Mensagens: 1400
Registrado em: Sex Jun 01, 2012 12:10
Formação Escolar: GRADUAÇÃO
Área/Curso: Engenharia Elétrica
Andamento: cursando

Re: valores das constantes a e b

Mensagempor Douglas16 » Dom Mar 31, 2013 22:44

Isso eu sei.
Mas considerando que a expressão do numerador pode ser fatorada como uma expressão quadrática em dois fatores:

Considerando X=sen\left(x \right), tenho que: 2X²+aX-(a+2)=(2X+a+2)(X-1) (expressão 1)

Logo vejo que o fator que possui a constante a, só zera quando a=-4 e usando este valor para encontrar o de b=3, sei que esse são os valores constantes, mas não sei como e porquê.
Para mim, o valor de a na expressão 1, deve ser tal que contenha o fator k^4, para eliminar a indeterminação.
Depois fazer a resolução para encontrar o valor limite.
Douglas16
Usuário Parceiro
Usuário Parceiro
 
Mensagens: 69
Registrado em: Seg Fev 11, 2013 19:15
Formação Escolar: ENSINO MÉDIO
Área/Curso: Matemática
Andamento: cursando

Re: valores das constantes a e b

Mensagempor e8group » Dom Mar 31, 2013 23:41

Na minha opinião ,sua solução não faz sentido ,qual finalidade de adotar este método ? Além do mais ,no denominador temos um polinômio enquanto no numerador não,portanto, não faz sentido a seguinte frase :

Douglas16 escreveu:Para mim, o valor de a na expressão 1, deve ser tal que contenha o fator k^4, para eliminar a indeterminação.
Depois fazer a resolução para encontrar o valor limite.



É isso ,caso dúvidas retorne .
e8group
Colaborador Voluntário
Colaborador Voluntário
 
Mensagens: 1400
Registrado em: Sex Jun 01, 2012 12:10
Formação Escolar: GRADUAÇÃO
Área/Curso: Engenharia Elétrica
Andamento: cursando

Re: valores das constantes a e b

Mensagempor Douglas16 » Seg Abr 01, 2013 02:16

Quanto ao polinômio, o correto é que eu teria de dizer: Deve-se eliminar a indeterminação \frac{0}{0}, eliminando ou assimilando por alguma identidade o termo {\left(x-\frac{\pi}{2} \right)}^{4}.

Agora veja se estou resolvendo corretamente:

Para que o limite exista:

(1) \lim_{x\rightarrow\frac{\pi}{2}} \left(-cos\left(2x \right)+a*sen\left(x \right)+b \right)=0 (equação 1)

Portanto 1+a+b=0, b=-\left(a+1 \right) (equação 2)

(2) Deve-se eliminar a indeterminação \frac{0}{0}, eliminando ou assimilando por alguma identidade o termo {\left(x-\frac{\pi}{2} \right)}^{4}.

Fazendo a substituição da equação 2 na equação 1:

-cos\left(2x \right)+a*sen\left(x \right)-\left(a+1 \right)= 2{sen}^{2}\left(x \right)+a*sen\left(x \right)-\left(a+2 \right)

Fazendo X=sen\left(x \right), tenho que:

2X²+a*X-\left( a +2\right)=(2X+a+2)(X-1) (equação 3)

Se X-1=sen\left(x \right)-1 e {sen}^{2}\left(x \right)-1=-{cos}^{2}\left(x \right) e -\frac{{cos}^{2}\left(x \right)}{{\left(x-\frac{\pi}{2} \right)}^{2}}\frac{1}{sen\left(x \right)+1}=\frac{{sen}^{2}\left(x \right)-1}{{\left(x-\frac{\pi}{2} \right)}^{2}}\frac{1}{sen\left(x \right)+1}

e de t=\left(x-\frac{\pi}{2} \right), portanto x=\left(t+\frac{\pi}{2} \right) (equação 4),

Portanto: -\frac{cos\left(x \right)}{x-\frac{\pi}{2}}=-\frac{cos\left(x \right)}{\left(x-\frac{\pi}{2} \right)}=-\frac{cos\left(t+\frac{\pi}{2} \right)}{t}=-\frac{sen\left(t \right)}{t}=-1

Assim (equação 3)/{\left(x-\frac{\pi}{2} \right)}^{4}, fica: [(2X+a+2)/{\left(x-\frac{\pi}{2} \right)}^{2}]*\left(-\frac{1}{sen\left(x \right)+1} \right)

E para que o limite exista 2X+a+2=0 quando x\rightarrow\frac{\pi}{2}, portanto a=-4 e b=3.

Portanto o limite é \lim_{t\rightarrow0} 2*{\frac{sen\left(t \right)}{t}}^{2}*{\left(\frac{1}{sen\left(x +1\right)} \right)}^{2}=\frac{1}{2}.

Concluindo: a=-4 e b=3. O valor do limite é \frac{1}{2}.
Douglas16
Usuário Parceiro
Usuário Parceiro
 
Mensagens: 69
Registrado em: Seg Fev 11, 2013 19:15
Formação Escolar: ENSINO MÉDIO
Área/Curso: Matemática
Andamento: cursando

Re: valores das constantes a e b

Mensagempor e8group » Seg Abr 01, 2013 13:31

Agora que observei que cometi um equívoco ,na verdade é (x - \pi/2)^4 e não (x - \pi/2)^2 ,fazendo k = x - \pi/2 fica no denominador k^4 e não k^2 .Caso fosse (x - \pi/2)^2 no denominador ,fixado b = -(a+1)  , \forall a\in \mathbb{R} o limite sempre existiria, como mostra o wolframalpha : http://www.wolframalpha.com/input/?i=li ... to++pi%2F2 , como não é o caso ,temos que impor mais condições sobre "a" e "b" .Peço desculpas pelo equívoco , parabéns pela dedicação a questão ,concluiu corretamente .
e8group
Colaborador Voluntário
Colaborador Voluntário
 
Mensagens: 1400
Registrado em: Sex Jun 01, 2012 12:10
Formação Escolar: GRADUAÇÃO
Área/Curso: Engenharia Elétrica
Andamento: cursando


Voltar para Cálculo: Limites, Derivadas e Integrais

 



  • Tópicos relacionados
    Respostas
    Exibições
    Última mensagem

Quem está online

Usuários navegando neste fórum: Nenhum usuário registrado e 22 visitantes

 



Assunto: Taxa de variação
Autor: felipe_ad - Ter Jun 29, 2010 19:44

Como resolvo uma questao desse tipo:

Uma usina de britagem produz pó de pedra, que ao ser depositado no solo, forma uma pilha cônica onde a altura é aproximadamente igual a 4/3 do raio da base.
(a) Determinar a razão de variação do volume em relação ao raio da base.
(b) Se o raio da base varia a uma taxa de 20 cm/s, qual a razão de variação do volume quando o raio mede 2 m?

A letra (a) consegui resolver e cheguei no resultado correto de \frac{4\pi{r}^{2}}{3}
Porem, nao consegui chegar a um resultado correto na letra (b). A resposta certa é 1,066\pi

Alguem me ajuda? Agradeço desde já.


Assunto: Taxa de variação
Autor: Elcioschin - Qua Jun 30, 2010 20:47

V = (1/3)*pi*r²*h ----> h = 4r/3

V = (1/3)*pi*r²*(4r/3) ----> V = (4*pi/9)*r³

Derivando:

dV/dr = (4*pi/9)*(3r²) -----> dV/dr = 4pi*r²/3

Para dr = 20 cm/s = 0,2 m/s e R = 2 m ----> dV/0,2 = (4*pi*2²)/3 ----> dV = (3,2/3)*pi ----> dV ~= 1,066*pi m³/s


Assunto: Taxa de variação
Autor: Guill - Ter Fev 21, 2012 21:17

Temos que o volume é dado por:

V = \frac{4\pi}{3}r^2


Temos, portanto, o volume em função do raio. Podemos diferenciar implicitamente ambos os lados da equação em função do tempo, para encontrar as derivadas em função do tempo:

\frac{dV}{dt} = \frac{8\pi.r}{3}.\frac{dr}{dt}


Sabendo que a taxa de variação do raio é 0,2 m/s e que queremos ataxa de variação do volume quando o raio for 2 m:

\frac{dV}{dt} = \frac{8\pi.2}{3}.\frac{2}{10}

\frac{dV}{dt} = \frac{16\pi}{15}